Can You Solve This Challenging Improper Integral?

Click For Summary
The discussion focuses on solving the improper integral of the function 1/(sqrt[x]*(1+x)) from 0 to infinity. The user identifies that the integral from 1 to infinity converges, as the function approaches zero, while the integral from 0 to 1 diverges due to the function approaching infinity. They attempt to use comparison tests to analyze the behavior near zero, concluding that the function behaves similarly to 1/(sqrt[x]) in that region. The user expresses confusion about handling the square root in the integration process and suggests using the substitution u = √x to simplify the problem. Overall, the thread highlights the challenges of evaluating improper integrals and the need for appropriate techniques.
RoganSarine
Messages
47
Reaction score
0
[Solved] Improper Integral Integration

Sorry, don't know how to use the latex stuff for integrals :P

Homework Statement



Integrate the following from 0 to infinity: 1/(sqrt[x]*(1+x))

Homework Equations



Integrate 0 to 1: 1/(sqrt[x]*(1+x))
Integrate from 1 to infinity: 1/(sqrt[x]*(1+x))

The Attempt at a Solution



Integrate from 1 to infinity: 1/(sqrt[x]*(1+x))
This is convergent because you can tell as it goes to infinity, it will approach 0.

So, I should (i think) be able to find the integral of this, but... I can't use improper fractions because that square root messes everything up (atleast, from my understanding how improper fractions work... When I try to break it up, the square root always ends up negative).

Integrate 0 to 1: 1/(sqrt[x]*(1+x))
This shoots off to infinity as the function approaches zero, so...

If b = 0
lim x -> b+ Integrate b to 1: 1/(sqrt[x]*(1+x))

If x gets really close to zero, I can assume
1/(sqrt[x]*(1+x)) ~ 1/(sqrt[x])

Therefore, by using comparison tests,

1/(sqrt[x]*(1+x)) \geq 1/(sqrt[x])

Since I know that (1/x^p) is convergent if p<1 for any bounds between 0-1.

Basically, I know the theory... I just got no idea how to solve the rest of it.
 
Last edited:
Physics news on Phys.org
Try the substitution u = √x
 
Question: A clock's minute hand has length 4 and its hour hand has length 3. What is the distance between the tips at the moment when it is increasing most rapidly?(Putnam Exam Question) Answer: Making assumption that both the hands moves at constant angular velocities, the answer is ## \sqrt{7} .## But don't you think this assumption is somewhat doubtful and wrong?

Similar threads

  • · Replies 7 ·
Replies
7
Views
2K
  • · Replies 105 ·
4
Replies
105
Views
6K
  • · Replies 14 ·
Replies
14
Views
2K
  • · Replies 10 ·
Replies
10
Views
2K
  • · Replies 12 ·
Replies
12
Views
2K
  • · Replies 22 ·
Replies
22
Views
3K
  • · Replies 13 ·
Replies
13
Views
3K
  • · Replies 5 ·
Replies
5
Views
2K
  • · Replies 5 ·
Replies
5
Views
2K
  • · Replies 2 ·
Replies
2
Views
2K